LSAT and Law School Admissions Forum

Get expert LSAT preparation and law school admissions advice from PowerScore Test Preparation.

 Brook Miscoski
PowerScore Staff
  • PowerScore Staff
  • Posts: 418
  • Joined: Sep 13, 2018
|
#66142
Mr. Mola,

The stimulus presents:

Self help books correlate with fewer doctor visits.
Better health correlates with fewer doctor visits.

Therefore, self help books improve health.

You are asked to identify the flaw, which is causal in nature, so you were correct to narrow it down to C and D. Knowing that this is a flaw question can help even if you have trouble stating the flaw. Looking above, we have a single effect and two causes. That is very much like answer choice (D). Answer choice (C), on the other hand, talks about having two effects instead of about having two causes. Since a correct flaw choice must describe the stimulus, (C) is wrong for the simple reason that it mixes up what we've got two of.

(C) also opens the door for the stimulus to be correct by suggesting that self help books could directly influence both health and doctor visits. That's not what we want a flaw choice to do--a flaw choice should illustrate why the argument could be wrong, not invent an explanation for why it could be right.

(D), on the other hand, explains that health and self help books could--independently of each other--correlate with doctor visits. So, a self help book could convince unhealthy people to avoid doctors without actually improving health.
 hope
  • Posts: 84
  • Joined: Feb 13, 2018
|
#93294
Hi All. Thanks for the explanations. Although I accept D as being the answer, I still have a problem with it. Here's why.

Someone's post used the abstract: A :arrow: C and B :arrow: C. But there is no A :arrow: B.

However, the stimulus itself states that there is a B :arrow: A where B is the self-help books and A is the improved health. Isn't that relationship established by the stimulus itself?

But AC D says NEITHER causally contributes to the other when ONE OF THEM actually does. (B :arrow: A)

Am I reading something wrong? Please let me know and thanks.
 Adam Tyson
PowerScore Staff
  • PowerScore Staff
  • Posts: 5153
  • Joined: Apr 14, 2011
|
#93490
That B :arrow: A claim you are talking about, hope, is the conclusion, which we should not accept as valid. The stimulus doesn't establish that it's true that having the book leads to better help. Rather, it improperly claims that having the book has that effect, when the evidence does not support that claim. Making that claim IS the flaw!

Answer D points that out by saying that it is possible that A doesn't cause B or vice versa, even though they both cause C. Not that one of them does not cause the other, but just that neither of them must cause the other.

Get the most out of your LSAT Prep Plus subscription.

Analyze and track your performance with our Testing and Analytics Package.